User Avatar
andrewnrabe705
Joined
Apr 2025
Subscription
Free
User Avatar

Sunday, Oct 21 2018

andrewnrabe705

Weaken Q's "Conclusion" v. "Argument" Semantics

I know that for weaken questions we're supposed to be critical of how the premise(s) support the conclusion (e.g. select the answer choice that "most weakens the argument" (an argument of course comprising one or several premises and a conclusion). But what if the question stem is cast in weaken form but refers only to the "conclusion"? For example, a stem might ask for which choice "casts the most doubt on the conclusion above." Is there a distinction to be drawn between these stems insofar as what the test is asking the test-taker to do? In other words, if I'm asked to weaken exclusively the conclusion, should I pay no attention to premises and select the answer choice that would simply weaken the conclusion, or do I need to without exception be cognizant of the premise-conclusion relationship?

PrepTests ·
PT149.S2.P3.Q14
User Avatar
andrewnrabe705
Wednesday, Nov 14 2018

For #14 I see that C is correct but I'm still having difficulty eliminating E. E questions whether A but not B discusses "the impact of social norms on creative output." JY emphasizes the 3 points in which author B states that the 3 norms operate like quasi-legal rights. But the rights are merely listed with no discussion of their consequence. I would think it is more natural to associate "impacts" with consequences and less natural to associate impacts with a statement that merely names the protection that the norms are thought to mirror.

Even if I concede that stating that norms mirroring legal protections amounts to a discussion of the impacts of social norms, there is the further problem that, from what I can glean from the text, author B does not include any specific discussion of the impact of the norms on creative output. Such discussion seems at best implied, and if so, something implied would not naturally seem to amount to something that passage B "discusses."

PrepTests ·
PT149.S3.Q2
User Avatar
andrewnrabe705
Wednesday, Nov 14 2018

I understand why the other answer choices are incorrect, but I was reluctant to choose E because it doesn't seem descriptively accurate. The conclusion is heavily qualified: "It is likely, therefore, that language developed primarily to facilitate animal domestication." The argument doesn't appear to be assuming that if something serves a purpose it "must" have developed to serve that purpose, as E suggests. Why is it that E is correct (aside from the fact that the other choices are less plausible) when it doesn't say that it is only "likely" (or the equivalent of likely) for the thing to have developed to serve a certain purpose?

#help

PrepTests ·
PT149.S3.Q3
User Avatar
andrewnrabe705
Wednesday, Nov 14 2018

C is clearly the best choice, but why doesn't the inclusion of "or all employers" push it out of the bounds of a necessary assumption and into the sufficient assumption category? For the argument to work (though further support would be needed) we don't seem to be required to assume that all employers use their employees as a means to their own ends.

#help

PrepTests ·
PT135.S4.Q26
User Avatar
andrewnrabe705
Tuesday, Nov 13 2018

C = The reasoning in the argument is flawed because the argument "assumes that, uh, you know, they're going to play volleyball"

PrepTests ·
PT148.S1.Q8
User Avatar
andrewnrabe705
Sunday, Nov 11 2018

I still don't see what C is preferable to E. To develop from small to large, three necessary conditions are listed: 1. spending less energy on finding food; 2. avoiding predators; 3. spending more energy on competition in mating

I see that C partially resolves the discrepancy by suggesting that otters will be able to spend less energy on finding food. So one of the three conditions is satisfied.

E seems to have the same effect. It suggests that the abalone in small form were able to avoid predators. Thus, like C, one of the three conditions is satisfied. I'm unsure why the point about selective pressure is relevant because the stimulus doesn't state that the large abalones flourished when they did come into existence. It only states that they did come into existence, and, under E's framework, perhaps attracted and were consumed by the otters. But the existing discrepancy (how it is that large abalones were able to come into existence given the three conditional requirements) seems roughly equally resolved by the two choices.

PrepTests ·
PT148.S1.Q3
User Avatar
andrewnrabe705
Sunday, Nov 11 2018

fun parse! (in BR of course)

PrepTests ·
PT143.S1.Q11
User Avatar
andrewnrabe705
Wednesday, Nov 07 2018

I'm confused as to why E can be so easily dismissed. It seems to me that it weakens the argument, though possibly tenuously.

The basis of the claim that crime fell by 20% due to the policing strategy turns in part on that strategy being implemented to concentrate on areas with the most crime. Choice E states that crime is more evenly distributed in the city in question than in other cities. Under this framework, couldn't you argue that it's (somewhat) less likely that a policing strategy that focuses on areas with the most crime will reduce crime in a noteworthy way?

D also seems to have no necessary impact on the argument as written. The police chief is not explicitly arguing that his strategy is effective insofar as being "good" at reducing crime. He is arguing that the drop in the crime rate is "the result of" his strategy. So notwithstanding the 20-30 disparity, that the national rate decreased by 30% doesn't seem immediately relevant. Rather, it arguably strengthens his argument: if his policing strategy is unique and yielded results different from the average, then perhaps the 20% (instead of 30%) drop was indeed the "result of" his policing strategy.

PrepTests ·
PT133.S1.Q21
User Avatar
andrewnrabe705
Friday, Nov 02 2018

Isn't E a flawed argument though?

The stimulus refers to calls on a twofold qualification, i.e. calls that are long-distance and on the Discount Phoneline. The answer choices parallel this twofold qualification by discussing not just university classes but university classes that "involv[e] extensive lab work."

E's conclusion seems to me not to parallel that qualification, because its conclusion only refers to "a university class" and we know nothing about university classes that do not involve "extensive lab work."

In real time I read through the answer choices and ended up guessing on another choice because E seemed flawed to me for that reason. When do we know that we can discount seeming inconsistencies like these on parallel questions?

Confirm action

Are you sure?